Which one of the following partners cannot have the third highest salary?

cglee on February 16, 2020

Homework Question #11

How could Q be third? It can't be but either can L be 4th? 1)H has to be first in this question 2)V>Q & L>J...with this I see the order being H,L,V,J,Q,S,P?

Replies
Create a free account to read and take part in forum discussions.

Already have an account? log in

shunhe on February 18, 2020

Hi @cglee,

Thanks for asking! I think the question I'm getting pointed to is the wrong one. I'm looking at PT #4 LG #1 Q#1 about the law firm game, which doesn't have the same characters that you're using. Which question are you asking about? Let me know and we'll try to get back to you.

Skylar on May 2, 2020

@cglee, happy to help!

We are not told that V > Q nor that L > J. This appears to be the cause of issue in your logic.

We should combine the chain from our original game setup with the specific requirement in this question that Q > L to get the following sequence:
> S
H > J > Q > L > V
> P

Now, let's go through our answer choices and eliminate those that must always be true.

(A) "H is first" must always be true, so it is eliminated. We see that H is the only variable that does not have anything that must precede it. In turn, it must precede the other six variables. So, H must be first.

(B) "L is fourth" is not necessarily always true, so it is the correct answer. From the sequence we designed above, our chain could start with either "H > J > Q > L" or with "H > J > Q > P." So, it is possible that P is fourth instead of L, which means that L is not always fourth.

(C) "V is not fourth" must always be true, so it is eliminated. As we established in the previous answer choice, the only two options for the fourth spot are L or P. Moreover, four variables need to precede V. Therefore, V will never be fourth.

(D) "J is not third" must always be true, so it is eliminated. Q will always be third, and J will always be second. This is because J is preceded only by H and must precede the remaining five variables.

(E) "Q is third" must always be true, so it is eliminated. As discussed above, Q is always third. It must be preceded by two variables (H and J) and must precede the remaining four variables. Therefore, it can only go third.

Does that make sense? Please let us know if you have any other questions!

Skylar on May 2, 2020

@cglee, I'd like to add that we are given the conditions V > Q and L > J in question #10. These conditions only apply to that question and not to later questions within the same game. My response addresses question #11, so those conditions are no longer in play.